LSAT and Law School Admissions Forum

Get expert LSAT preparation and law school admissions advice from PowerScore Test Preparation.

User avatar
 Dave Killoran
PowerScore Staff
  • PowerScore Staff
  • Posts: 5850
  • Joined: Mar 25, 2011
|
#27057
Complete Question Explanation
(The complete setup for this game can be found here: lsat/viewtopic.php?t=3418)

The correct answer choice is (B)

This is the most difficult question of the game. The difficulty arises from the wording of the question and the nature of the answer choices. Reworded, the question asks, “Of the people selected to attend the retirement dinner, there must be at least one from which one of the following pairs?” or, alternately, “Which one of the following contains a pair of people who cannot both be eliminated from attending the retirement dinner?”

The two easiest answers to this problem would be J or K, or N or P, because we know from the first two rules that at least one person from each of those pairs must be selected. Of course, because those answers are so obvious, they do not appear among any of the lettered answer choices. Instead, to find the solution to this question, we must return to the inference discussion in the setup. In the discussion of the selection pool, we noted that exactly two of L, M, and Q must be selected, and if one of those three variables is not selected, then the other two must be selected. Thus, no two of those variables can be eliminated from contention. Answer choice (B), which contains L and M, contains a pair of variables where one or both must be selected, and thus answer choice (B) is correct.

To consider this problem from another angle, if you remove both L and M from the retirement dinner, you will not be able to come up with a group of four people that also meets the requirements of the rules.
 karlieg
  • Posts: 1
  • Joined: Oct 04, 2014
|
#17028
Would someone be so kind as to introduce how they solved question #9? Thank you.

-Karlie
 Ron Gore
PowerScore Staff
  • PowerScore Staff
  • Posts: 220
  • Joined: May 15, 2013
|
#17029
Hi Karlieg!

To cite Dave from a response about #11 from this same game:
Dave Killoran wrote:Because J, K, N, and P will collectively occupy exactly two of the spaces (this is known from the first two rules), the remaining two spaces are occupied by the group of L, M, and Q. Thus, we can infer that two people from the group of L, M, and Q must always be selected. Note that any time one of the members of the group of L, M, and Q is not selected, the other two must be selected.
So, given the fact that two people from L, M and Q must always be selected, we can choose answer choice (B). To test this answer choice, imagine the situation if neither L nor M were selected. We would have only Q of the group of L, M and Q, although we just said we've got to have two of the three at all times.

If you test the other answer choices in the same way, you'll see that no other answer choice leads to a similar violation of the rules. For example, you can have a valid template that includes neither K nor Q (testing answer choice (A)).

Please let me know if that helps.

Thanks,

Ron

Get the most out of your LSAT Prep Plus subscription.

Analyze and track your performance with our Testing and Analytics Package.